14
$\begingroup$

Given two convex bodies $A$ and $B$, in $\mathbb R^3$ let's say. We define $A(t)$ and $B(t)$ as $A+xt$ and $B+yt$ where $x,y$ are two arbitrary points. (That is the Minkowski sum, so the two bodies are moving at constant velocity in the $x$ and $y$ directions, and $t$ is the time variable.) Can one show that the function $$f(t)=\operatorname{Vol}(A(t)\cap B(t))$$ is unimodal? That is nondecreasing up to some point, and then nonincreasing.

$\endgroup$
2
  • $\begingroup$ Gjergji, I suspect that you ask for the 3D version because this is known in 2D. Can you give a link to the corresponding result? $\endgroup$ Jun 4, 2010 at 10:28
  • 1
    $\begingroup$ I suspect the proof, even in the general case, not to be very hard, but I don't have any reference. I was discussing Kneser-Poulsen with a friend today, and thought that the question above is one of the simplest one could start asking in a series of related questions. $\endgroup$ Jun 4, 2010 at 11:13

1 Answer 1

16
$\begingroup$

The sets $\{ (A(t),t)|t\in \mathbb{R} \} \subset \mathbb{R}^4$ and $\{ (B(t),t)|t\in \mathbb{R} \} \subset \mathbb{R}^4$ are convex, their intersection $K$ is a bounded convex set, and $f(t)$ is the volume of the slice of $K$ at height $t$. By Brunn-Minkowski inequality, this is log-concave, so definitely unimodal.

$\endgroup$
5
  • $\begingroup$ Thank you Thorny! This is nice. By the way, you mean $f(t)^{1/3}$ to be concave right? Unimodality follows in any case. $\endgroup$ Jun 4, 2010 at 12:30
  • $\begingroup$ If $f^{1/3}$ is concave then $f$ is log-concave. Thorny's kind of formulation is frequently preferred when it is possible, because it's dimension-free. $\endgroup$ Jun 4, 2010 at 13:35
  • $\begingroup$ The intersection $K$ need not be bounded. Consider the case $A \cap B \ne 0$ and $x = y$. $\endgroup$ Jun 4, 2010 at 13:46
  • 1
    $\begingroup$ See also Zalgaller, V. A., On intersections of convex bodies. $\endgroup$ Jun 4, 2010 at 13:50
  • $\begingroup$ @Mark Meckes: You are right, I think I misread log-concave for concave for some reason... $\endgroup$ Jun 4, 2010 at 13:53

Your Answer

By clicking “Post Your Answer”, you agree to our terms of service and acknowledge you have read our privacy policy.

Not the answer you're looking for? Browse other questions tagged or ask your own question.